How to determine the reality of mystical experiences?

  • Thread starter PIT2
  • Start date
  • Tags
    Reality
In summary, the best way to determine the reality of mystical experiences is to learn how to achieve them through dedicated practice, and to continue experiencing them until a sense of certainty is realized. While sense of reality is often used as a heuristic for judging the reality of an experience, it is not a foolproof indicator as it can be generated by the brain even in situations where the experience is not actually real. Additionally, the brain signatures of experienced meditators are different from those of the average human, indicating that these experiences may give a glimpse into a deeper level of reality. Ultimately, the best way to determine the reality of these experiences is through personal experience and continued practice.
  • #141
RVBuckeye said:
DMuitW,
Since you have been following this thread for a long time, I'm sure you've noticed that I've been arguing the physicalists position. While all your points are interesting, no doubt, it seems the only part of the NDE that could provide the most answers to the existence of the bridge between the different realities is the OBE.

However, all this leaves room for other posibilities. Can OBE's be studied independently of the NDE? In other words, it would be unethical to induce brain death for the purpose of providing data on OBE's that occur during an NDE. So the other option is inducing OBE's without brain death such as by external means. But would everyone agree that it is the same state? If there was never again a person like Mrs. Z., who could read a 5 digit number off a card in another room, would that end the debate? No. There are people that would say, "It must be different than one experienced in a NDE."

Also, I would be interested to know your take (or possibly Pim Van Lommel's) on why not everyone experiences an NDE or an OBE in the first place. This part alone leaves room to explore the psychological factors. Which is why I find the sleep studies just as relevent.

Standalone OBE’s can be studied separately from NDE’s. Many experiences have been provided but have been reported to occur mostly spontaneous. Only few cases report inducing them at will. This could be a path to follow in the research of the reality of these OBE’s.

I should note that OBE’s that are accompanied with other parts of the NDE’s and standalone OBE’s are two very different things. OBE’s with other parts of the NDE’s are the result of a highly stressing and life threatening event whereas OBE’s that don’t go accompanied by other parts of the NDE’s lack this feature and differ qua interpretation of their contents.
It would however be logic to assume that both kinds of OBE occurrences fall back on the same mechanism. I don’t even contest that induced experiences (I won’t call them NDE’s to avoid misunderstandings) through Ketamine and such could (partially) rely on this mechanism (on which more a little further in this text).

Why not everybody who ends up in a life threatening event experiences an NDE could be explained by different factors.
Firstly, noted by Van Lommel as well is that in order to remember an NDE, having a good short term memory seems to be essential. Patients with memory defects after prolonged resuscitation reported fewer experiences than others in the study. This can be accompanied by forgetting the experience. To quote Van Lommel: “at 2-year follow-up, two patients remembered a core NDE and two an NDE that consisted of only positive emotions that they had not reported shortly after CPR, presumably because of memory defects at that time.”

Furthermore I add that many patients psychologically repress their experience in order not to be declared mentally ill. This is a real fear that initially bothered a majority of the patients that have come over the bridge with their experience.

Secondly - and presumably more fascinating for physics – is the mechanism that is responsible for this assumed delocalized mind. When we consider our worldly reality and another implicate reality that may be out there, there should be a mechanism responsible for the connection between these two. When we consider NDE’s, it seems logic to think that this mechanism at a certain moment “lets go” the physical part of which could be connected to the brain.
I note that there are strong indicators that this mechanism doesn’t solely rely on “the physical moment of brain death”, but also may depend on a subjective decision that is in the coextensive reality out there (and therefore is inaccessible to scientific objectivism) . This is backed up by my earlier assumption that standalone spontaneous OBE’s should logically use the same principle of delocalizing the mind, and obviously don’t require brain death. Also, NDE’s occur in comatose patients, who cannot be brain dead as they lived to tell.

The implication of this is that pinning down an exact moment where an NDE or OBE occurs is hard. If indeed there is a physical counterpart that is wired to the brain, it may be possible to detect this, but this may as well be refuted by the apparent “subjective” occurrences of OBE’s that were assumed to fall back on the same principle of a delocalized mind.

Furthermore, if we look at the possibility of a delocalized mind, it should not be neglected that NDE’s pose serious ethical questions to euthanasia in comatose patients as there is no evidence that the patient is no longer in some form “conscious”.
 
Physics news on Phys.org
  • #142
Paul Martin said:
In my posts, I usually use the radio analogy only to point out my views on the brain/mind separation and the mechanism for communicating between them. When it comes to speculating on the "beings" which inhabit another world and their relationships to physical biological bodies I typically use the more complex analogy of a Mars rover.

I already foresaw that from your arguments, since communication between mind and body occurs both ways, but I still think the question needs to be addressed.

The way I see it, mind-to-mind communication should be as simple as mind-to-body-to-body-to-mind, unless the mind is restricted to the body (ie, unless physicalism is true). To use your analogies, it's very easy for two JPL scientists to talk to each other, all they have to do is meet. Same for two radio show hosts. You seem to be saying there's something in this extra dimension in which consciousness exists which prevents mind-to-mind communication; that fits the facts, but it sounds like an ad-hoc, after-the-facts explanation. The problem is that there is no reason to postulate that it's more difficult for consciousness to move around in this extra dimension than it is here.

"How come it is easier for the JPL scientist to communicate with the rover than it is for two rovers to communicate between themselves?"

My question was really "how come it is easier for the JPL scientist to communicate with the rover than it is for two JPL scientists to communicate between themselves?".

The answer is that in the grand scheme of things, communication links were specifically designed between JPL and the rover and such links were not designed for inter-rover communication.

Since my question was not clear, let me venture as to what your adjusted answer would be. You would probably say that, in the grand scheme of things, inter-consciousness communication links were not designed. If that is the case, then I really hope the physicalists are right, as spending an eternity in absolute loneliness must be a frightful thing.

By contrast, telepathic communication would require the path 'mind --> mind' which would occur completely in the "other world". At this point, we can only further speculate on our speculation. My guess is that communication among entities in the "other world" must be difficult.

What if it is only difficult when beings in that "other world" are busy with affais of this one? For instance, it's extremely difficult to communicate with a teenager listening to loud rock-and-roll. If your theories are true, I'd think it possible to shut off the loud music and have some sort of telepathical communication.

(then again, sometimes it's impossible to communicate with a teenager at all, even in the absence of loud music. Langauge difficulties may be a problem, as you mentioned)

I just think that scientists would be more willing to accept the reports from one of their own credentialed people than they would from you, me, or other "unqualified" observers.

Actually, what happens when a "qualified" observer tells things scientists don't like to hear, instead of paying attention they simply disqualify the person. I've see it happen quite often; contrary to materialist propaganda, scientists are people too and can often be extremely close-minded.

It is the scientists who need to be convinced, not you or me.

I see the problem in a different way. I think the ordinary person is divided between the opinions of the scientists and all other competing ideas, but is more strongly influenced by the scientific position due mostly to propaganda and censorship. We live in sort of a materialist dictatorship, except the dictators use intellectual coercion methods rather than physical ones.

I'm not sure where the pressure for materialism comes from. My guess is that the elite will force the ideology that keeps them in power, so in theocracies the dominant philosophy is some form of religion, whereas in capitalism the dominant philosophy is one that stimulates people to "enjoy life to the fullest" (meaning "spending as much money as you possibly can")

So I do think it's actually the people who need to be convinced. They are the ones abandoning traditional beliefs held for millenia. The materialists will always be around.

You are right; I have never tried it myself and I don't have much to go on. Two examples did make me wonder though.

I have also seen cases of "spirit communication". I think if you dig through the overwhelming amount of charlatanism, you do find a lot of very interesting and intriguing stuff. As I said before, those have been a part of folk knowledge for a long, long time. It's not like there's something new to be discovered.

You mean they have got to be kidding. I agree with you, remember?

I wrote "you got to be kidding" as an expression of amazement. I know we agree.

Something that struck me as interesting was that both Seth and Uncle Darrell frequently interjected remarks about how difficult it was for them to express their concepts in English. It is the serial nature of the language which gave them the most problem.

I totally sympathize with them :smile:

Now seriously, I don't really believe disembodied spirits talk to humans, unless perhaps to deceive them. It's interesting that many religions advise not to play with this kind of stuff, that the messages "come from the devil", and I think it makes some sense. If you put yourself in the position of those "spirits", you'd probably think it does no one any good giving out cryptic messages that most likely will be misinterpreted.

What I really believe is that those messages come from places in our own minds to which we have difficulty gaining access, except in altered states of consciousness. You may be impressed with Seth's writings, but the fact is that you have the power to see how intelligent they are, so you must have the same intelligence yourself. It goes even further; if you appreciate great works of art, say a masterful painting or a great work of literature, that means you have what it takes to understand it. We are not much less talented than Van Gogh or Tolstoy if we have the ability to see their works as they did. We only lack, perhaps, the ability to make creative use of our talent.

So with all those "messages from spirits", I think you can easily see that going through an altered state of consciousness often unlocks powers of the mind that are normally inaccessible. That is, Seth and Jane Roberts are the same person, except Seth is hidden inside Jane's mind and only comes out when Jane's personality gets out of the way, so to speak.
 
  • #143
Dichter said:
You seem to be saying there's something in this extra dimension in which consciousness exists which prevents mind-to-mind communication; that fits the facts, but it sounds like an ad-hoc, after-the-facts explanation. The problem is that there is no reason to postulate that it's more difficult for consciousness to move around in this extra dimension than it is here.
You are right; my explanation was not satisfactory. It needs yet more refinement. I think the stumbling block here is confusion in the identification of an individual human being. My view makes this identification much more complicated than it is in a materialistic view where the individual is clearly identified simply as the biological body.

In my view, an individual is an ordered pair, so to speak. It is the Cartesian pair of body and mind where each component resides in a separate "world". So when we talk about two humans communicating, we have to be clear about which of the four components are involved.

I agree that it should be easy for two minds to communicate directly, and for all I know, it is. But in order for the bodies to report on that communication here in the physical world, then the bodies must also be involved.

My guess is that the difficulty of achieving telepathy between human bodies stems from the circumstances of the minds and what is going on in that mental world. Yes, two JPL scientists or two talk show hosts can talk easily if they have the intention to talk and they have some sort of communication link. But in the circumstance where one or both JPL scientists are busy driving their respective rovers, or the circumstance where one or both talk show hosts are busy conducting their respective shows, it is not likely that they would be able to talk to their counterpart simply because they are preoccupied.

As you suggested with your teen-ager scenario, it may be easier to achieve telepathy if the mind can withdraw from the preoccupation with the body, for example while the body is meditating or sleeping. I suspect that such communication goes on during sleep, but the communication is not encoded into English and not stored in the brain so upon awakening, the body can't accurately or completely report on what transpired. Those are just some of my guesses as to why telepathy is difficult and unreliable, which it obviously is.
Dichter said:
Quote: [Paul]
"How come it is easier for the JPL scientist to communicate with the rover than it is for two rovers to communicate between themselves?"

My question was really "how come it is easier for the JPL scientist to communicate with the rover than it is for two JPL scientists to communicate between themselves?".
In order to be seen as telepathic communication between humans it would have to be the rover to rover case. If there is communication strictly between the minds, then that would be the JPL scientist to JPL scientist case but it would not be reportable by the rovers (humans).
Dichter said:
Since my question was not clear, let me venture as to what your adjusted answer would be. You would probably say that, in the grand scheme of things, inter-consciousness communication links were not designed. If that is the case, then I really hope the physicalists are right, as spending an eternity in absolute loneliness must be a frightful thing.
I can see that I have confused things. I said that "such links were not designed for inter-rover communication", meaning telepathic links. Of course inter-rover communication links were designed (or at least they exist for sure) but they involve acoustics and they operate strictly in the physical world. I think that in the grand scheme of things, inter-consciousness communication links operate up and down the hierarchy and they may very well operate horizontally in each level also, as they do down here in our physical level. In any case, I think the prospects for eternity are good, especially if the physicalists are wrong.
Dichter said:
Actually, what happens when a "qualified" observer tells things scientists don't like to hear, instead of paying attention they simply disqualify the person. I've see it happen quite often; contrary to materialist propaganda, scientists are people too and can often be extremely close-minded.
I agree, but I still think that route is our only hope. There will be a few obstinate daring scientists who will doggedly pursue what they believe to be a new paradigm in spite of being disqualified, shunned, unfunded, or derided. The eventual success of plate tectonic theory, and the eventual acceptance of the bacterial cause for ulcers, are prime examples of how this can happen. Myriad non-scientists proposed continental drift for years and got nowhere. Myriad non-scientists could have deliberately infected themselves with Helicobacter pylori and cured the resulting ulcers with antibiotics and the scientists would have dismissed the evidence as crackpot folklore. In each case, it took a daring "qualified" scientist to buck the dogma and force a paradigm change.
Dichter said:
I see the problem in a different way. I think the ordinary person is divided between the opinions of the scientists and all other competing ideas, but is more strongly influenced by the scientific position due mostly to propaganda and censorship. We live in sort of a materialist dictatorship, except the dictators use intellectual coercion methods rather than physical ones.
Yes, we do see this somewhat differently. I think the overwhelming influence on the public to accepting the scientific position is the undeniable success of science in solving so many real human problems. I think this is positive and well deserved. I don't think we should turn away from science as the authority, but instead we should encourage an extension to the scope and methods of science so that they can take on even more interesting or important problems.
Dichter said:
I'm not sure where the pressure for materialism comes from.
I think it comes from their spectacular and unqualified success -- so far.
Dichter said:
Now seriously, I don't really believe disembodied spirits talk to humans, unless perhaps to deceive them. It's interesting that many religions advise not to play with this kind of stuff, that the messages "come from the devil", and I think it makes some sense. If you put yourself in the position of those "spirits", you'd probably think it does no one any good giving out cryptic messages that most likely will be misinterpreted.
I agree completely. I think it is presumptuous of us to think we know what might be going on in the putative spirit world. There seems to be ample evidence that it doesn't do us much, if any good, to play around with that kind of stuff.
Dichter said:
What I really believe is that those messages come from places in our own minds to which we have difficulty gaining access, except in altered states of consciousness.
I agree with you completely here, but I suspect we interpret your statement quite differently -- that is if you don't accept my single-mind hypothesis.

In my view, when you talk about "places in our own minds", you imply that "minds" are the possessions of the antecedent of "our". Now what exactly do you mean by "our"? Who is the real owner of the mind? Since your body, with its fingers, was the thing that actually typed your post that I am reading, it seems natural to think of "us" as being human bodies, and that we "own" minds which reside someplace. That's OK, and it makes communication among us a little easier, but in my view things are a little more complicated.

If cars could talk (and believe), your statement would be tantamount to a car saying, "What I really believe is that those messages my driver is using in order to navigate and control where I am going, come from places in (or available to) my driver to which I have difficulty gaining access, except in altered states of consciousness." But cars can't talk, or believe, or experience consciousness, altered or otherwise. In my view, human bodies can talk, but they can't believe, or experience consciousness. Those things go on in the mind which is separate from the body. (As you can obviously tell, I am a Cartesian dualist. I may be the last one, but I think Cartesian duality explains much more with respect to mental phenomena than any other scheme.)

So when we say "I believe", who is the "I" making the claim? Is it the mind or the body? I say it is the mind. But who is talking, or typing? It is obviously the body. Materialists don't have to worry about this distinction, but in my scheme it is very important to keep things straight or confusion sets in very quickly.
Dichter said:
You may be impressed with Seth's writings, but the fact is that you have the power to see how intelligent they are, so you must have the same intelligence yourself. It goes even further...
Indeed it does go further -- at least in my view. In my view, the ultimate mind of Seth is exactly the same as the mind "in me" or that "I" use. It is also the same one "driving" you, and everyone else. So it should not be surprising that by looking inward into our mind we can discover how intelligent we are. It's just that with the preoccupation of driving a body through the process of living a biological life, we have temporarily forgotten most of what we know, as Socrates pointed out long ago.
Dichter said:
So with all those "messages from spirits", I think you can easily see that going through an altered state of consciousness often unlocks powers of the mind that are normally inaccessible. That is, Seth and Jane Roberts are the same person...
Yes, I can easily see that and I do. But I would change your last sentence here to say that Seth and Jane Roberts share the same mind. With the right definition of "person" I would agree that they are the same person.
Dichter said:
...except Seth is hidden inside Jane's mind and only comes out when Jane's personality gets out of the way, so to speak.
I take exception to this exception. It sounds too materialistic to me. I would say that Seth has taken over Jane's body in order to use it as part of the communication link to report in the physical world some ideas Seth has in the other world. Jane's personality needs to get out of the way in order for this to work -- so to speak (good one Dichter :)). And that, incidentally, is exactly what Seth said is going on also.

As always, it's fun talking with you, Dichter.

Paul
 
  • #144
Paul Martin said:
As you suggested with your teen-ager scenario, it may be easier to achieve telepathy if the mind can withdraw from the preoccupation with the body, for example while the body is meditating or sleeping. I suspect that such communication goes on during sleep, but the communication is not encoded into English and not stored in the brain so upon awakening, the body can't accurately or completely report on what transpired. Those are just some of my guesses as to why telepathy is difficult and unreliable, which it obviously is.

Some cool experiments have been done and duplicated in the last few years which seem to indicate that such a "telepathic" connection exists (though they call it "neural energy transfer" or "correlated brain signals").

Here is one:

Replicable functional magnetic resonance imaging evidence of correlated brain signals between physically and sensory isolated subjects.

University of Washington, Seattle, WA.

Conclusions:
These data replicate previous findings suggesting that correlated neural signals may be detected by fMRI and EEG in the brains of subjects who are physically and sensorily isolated from each other.

http://www.ncbi.nlm.nih.gov/entrez/..._uids=16398586&query_hl=1&itool=pubmed_docsum

And:

Electroencephalographic evidence of correlated event-related signals between the brains of spatially and sensory isolated human subjects.

Bastyr University/University of Washington Consciousness Science Laboratory, Bastyr University, Kenmore, WA 98028, USA.

The results indicate that in some pairs of human subjects a signal may be detected in the brain of a distant member of the pair when the brain of the other member is visually stimulated.

http://www.ncbi.nlm.nih.gov/entrez/...ve&db=PubMed&dopt=Abstract&list_uids=15165411

The best results were seen in pairs of subjects that were emotionally connected to each other, or when they had meditated together prior to the test.
 
  • #145
Thanks PIT2,

Those studies seem to confirm my assessment that telepathy is difficult and unreliable. But I am delighted to see that qualified scientists are putting effort into trying to understand consciousness related mysteries.

Paul
 
  • #146
Interesting points, Paul. You made me realize how difficult it is to let go of old ideas in order to understand new ones.

Paul Martin said:
In my view, an individual is an ordered pair, so to speak. It is the Cartesian pair of body and mind where each component resides in a separate "world". So when we talk about two humans communicating, we have to be clear about which of the four components are involved.

I have no problem with the Cartesian view, the problem is thinking of a world which cannot be expressed using concepts from our language.

But in order for the bodies to report on that communication here in the physical world, then the bodies must also be involved.

That is an interesting perspective which never occurred to me.

I have noticed a strange phenomenon when I wake up in the morning; I don't know if it applies to me only, I believe everyone is the same. Many times I wake up thinking exactly the same thought I had when I went to bed, but my mood can be completely different. That led me to think I go through experiences at night (dreams) during which I'm not thinking, but the emotional outcome of those experiences still stay with me. So even though I don't remember the dreams, I'm still affected by them on a subconscious level.

This raises an interesting question for me: what is the relationship between the self and the subconscious mind? Can we become conscious of what is now within the subconscious? If we can, can we describe what we find using ordinary language?

As you suggested with your teen-ager scenario, it may be easier to achieve telepathy if the mind can withdraw from the preoccupation with the body, for example while the body is meditating or sleeping. I suspect that such communication goes on during sleep, but the communication is not encoded into English and not stored in the brain

If it's not stored in the brain, is it stored at all?

In order to be seen as telepathic communication between humans it would have to be the rover to rover case. If there is communication strictly between the minds, then that would be the JPL scientist to JPL scientist case but it would not be reportable by the rovers (humans).

Fascinating!

In any case, I think the prospects for eternity are good, especially if the physicalists are wrong.

I'm not afraid of eternity, I was just trying to point out that those horizontal channels of communication must exist.

There will be a few obstinate daring scientists who will doggedly pursue what they believe to be a new paradigm in spite of being disqualified, shunned, unfunded, or derided. The eventual success of plate tectonic theory, and the eventual acceptance of the bacterial cause for ulcers, are prime examples of how this can happen.

If history serves as a guide, you are certainly right. Still, I cannot imagine such a turn of events in the near future. A scientific proof of the immaterial nature of consciousness would do as much damage to materialism as evolution theory did to religion; it would shake the foundations of modern society. I am not sure what could possibly follow; I suspect it would mean the end of the war between science and religion, and religion would have won. Which is why I don't think it will happen anytime soon.

I think the overwhelming influence on the public to accepting the scientific position is the undeniable success of science in solving so many real human problems.

If that is the case, why do we still have fundamentalists around? Fundamentalists are witness to those successes as much as anyone else, yet they seem unmoved by them. It seems to me it's mostly people with post-secondary degrees that care that much about science.

So when we say "I believe", who is the "I" making the claim? Is it the mind or the body?

I honestly don't know the answer to that question. Certainly the sound of the words "I believe" comes from my mouth. What about knowledge of the words themselves? Does the body know English, or is that in the mind? I'd be inclined to believe language resides in the brain, but in that case what is there to an idea beyond its expression in language?

I take exception to this exception. It sounds too materialistic to me.

The real problem with all this stuff is to keep our feet on the ground. Forget the propaganda you keep hearing about science, that it is based on empirical methods or peer review or whatever; the thing that really allows people to achieve scientific success is having both feet on the ground. Now that doesn't mean we can't aim as high as possible, only that we should not embark in flights of fancy.

The problem with materialism is that much of it is true. We can't dismiss an explanation just because it is too materialistic, we can only dismiss it if it's not true.

I would say that Seth has taken over Jane's body in order to use it as part of the communication link to report in the physical world some ideas Seth has in the other world. Jane's personality needs to get out of the way in order for this to work

(I wrote a comment on this, and then deleted it because I realized I was writing out of prejudice. For personal reasons I am a bit scared of this "talking to spirits" thing; I've known quite a few people who have been negatively influenced by those ideas, in one case to the point of bringing unhappiness to a family. No matter how I try, I can't be objective about this)
 
  • #147
Dichter said:
I have noticed a strange phenomenon when I wake up in the morning; I don't know if it applies to me only, I believe everyone is the same. Many times I wake up thinking exactly the same thought I had when I went to bed, but my mood can be completely different.
Have you thought about this possibility: Could it be that you (the conscious, thinking part of you) are waking up in a different body than the one you were in when you went to sleep? If the brain were loaded up with memories from the previous occupant, you would wake up thinking and feeling that there was continuity of identity across the interval of sleep. That could be an illusion. The different mood you described could be explained by different mood-inducing hormones that might have been released during the night, or it could be that your mood is different because you are a different driver from the one on the previous day. I think that there is no way to prove that you are the same person (i.e. conscious entity) that you were yesterday. When I wake up, I typically feel a few milliseconds of disorientation as I put together who and where I am. Sometimes, if I am traveling, this even takes seconds. I think the discontinuous nature of consciousness is a feature that will prove to play a prominent role in the true explanation of consciousness when we finally do figure it out.
Dichter said:
This raises an interesting question for me: what is the relationship between the self and the subconscious mind? Can we become conscious of what is now within the subconscious? If we can, can we describe what we find using ordinary language?
Ahead of these questions is the interesting question of what, if anything, is the subconscious mind in the first place. Since I don't believe the conscious mind is seated in the brain, it doesn't seem likely that the subconscious mind is either. I suspect that what we call 'the subconscious mind' is a collection of disparate functions some of which reside in the brain and others of which are outside the brain with the conscious mind. Functions like the regulation of endocrine secretions or the autonomic nervous system I would expect to be located in the brain similar to the on-board computers in a Mars rover. These functions may or may not be attributed to the subconscious mind depending on personal preference -- or maybe there is an official position taken by psychologists. I just don't know. But the subconscious function of providing AHA! types of insight I suspect take place outside the brain. Those, of course are just guesses.

In any case, from my point of view, there is only one mind in existence, so there is only one self. That self transcends and subsumes all the physical bodies. While this mind is driving a body, it is obviously limited in its available knowledge. In this condition it is convenient to recognize the mind/body pair as an individual, and it is also convenient to ascribe selfhood to the pair. And so the answers to your questions about the relationships depend on exactly what you mean by 'self', 'mind', 'subconscious', 'consciousness', 'we', etc. But I have been through this before. I think we can describe these relationships using ordinary language as long as we are clear about the connotations of the words I just listed.
Dichter said:
If it's not stored in the brain, is it stored at all?
I think so. I think that most memory, in particular all long-term memory, resides outside the brain (and outside the physical universe). This would explain why the recollection of an entire lifetime of experiences can be immediately available during an NDE. It would also explain why scientists have not been able to figure out where in the brain memories are stored.
Dichter said:
If history serves as a guide, you are certainly right. Still, I cannot imagine such a turn of events in the near future. A scientific proof of the immaterial nature of consciousness would do as much damage to materialism as evolution theory did to religion; it would shake the foundations of modern society. I am not sure what could possibly follow; I suspect it would mean the end of the war between science and religion, and religion would have won. Which is why I don't think it will happen anytime soon.
If I were a betting man, I would bet on the other horse. I think a true breakthrough in the understanding of consciousness will happen in the near future. Surprising things happen quickly these days.

Yes, it will shake the foundations of modern society, but those foundations have been shaken before. We will survive. In my opinion, much more damage will be done to religious foundations than will be done to scientific foundations. The scientists will only have to admit that there is something going on in addition to what they have already discovered and described. That won't be any more of a problem than dark matter or dark energy. The religionists, on the other hand, will find out that their cherished notions of an infinite, almighty God who takes their side in war is in fact a Wizard of Oz with powers that are limited and finite. In fact, to paraphrase Pogo, they will find out that "He is us". I think that is good news though. The acceptance of the new view will once and for all pull the rug out from under the fundamentalists and hopefully their malevolent Jihads and crusades will cease.
Dichter said:
If that is the case, why do we still have fundamentalists around? Fundamentalists are witness to those successes as much as anyone else, yet they seem unmoved by them.
Their heads are in the sand and their days are numbered.
Dichter said:
I'd be inclined to believe language resides in the brain, but in that case what is there to an idea beyond its expression in language?
I believe part of the language function resides in the brain. I think Chomsky identified those functions. But I think all cognition, awareness, understanding, and other aspects of consciousness, reside outside the brain. I think there is a great part of an idea that is beyond its expression in language. In fact I think all language expressions are inadequate to fully capture any idea. Take the word 'triangle' for example. The word expresses an idea which we can easily apprehend, but think of the paragraphs of language it would take to describe one exactly with their perfectly straight sides, made of ultra-thin yet not vanishing lines, meeting precisely to form the super sharp corners, etc., etc., etc. In the big scheme of things, I think that language, as powerful as it is, is very crude and limited in its potential. Ideas make language pale by comparison.
Dichter said:
The real problem with all this stuff is to keep our feet on the ground.
You may have noticed that I have that problem. My flights of fancy try to take me all sorts of places. My strategy though, is to tie the lead weights of sensibility to my ankles to try to keep my feet on the ground. For any wild idea I consider, I always accept it only if it makes sense to me. I think reality makes sense, so I am disinclined to accept any explanation for anything that doesn't make sense. My ideas may not make sense to others, but they all make sense to me. I am always eager to hear from anyone who thinks that my ideas don't make sense. If, after discussing an idea, someone can show me the nonsense in my ideas, I will abandon the idea immediately with no regrets. This is an open invitation to anyone reading this. Please tell me where and why you think my ideas don't make sense.
Dichter said:
For personal reasons I am a bit scared of this "talking to spirits" thing
Me too.

Paul
 
  • #148
Paul Martin said:
Have you thought about this possibility: Could it be that you (the conscious, thinking part of you) are waking up in a different body than the one you were in when you went to sleep?

I must confess that idea is way too advanced for me. You probably mean something completely different from what I understood so it's hard for me to comment.

But I'd like to propose a little twist to this dialogue. Since the central issue is the nature of the mind, whether it is physical or not, I think there must be a better approach than mere speculation. I also think that if the mind is not physical, then there's nothing science can do about it. If the mind is really non-physical, science will be forever constrained to postulate unknown physical processes as the cause of physical phenomena that are not well understood. Such already is the situation with all fields associated with biology.

If the mind is non-physical, I think it's not the scientists who will prove that but the mathematicians. The other alternatives are philosophers and mystics, but the former are too sloppy in their definitions and the latter are not trustworthy. Here is how I see the non-physical nature of the mind could be approached from a mathematical perspective:

We'd have to start with a clear mathematical definition of "physical". I'm not a mathematician, but as far as I know such a definition does not yet exist. I don't think it should be difficult though; we do not necessarily have to know exactly what "physical" is, all we have to do is come up with a definition that is consistent with everything we know.

I would venture that a mathematical definition of "physical" would imply that nothing physical can have infinite or zero magnitude. Then the next step would be to assert that one or more features of consciousness cannot be physical by that definition. Now the interesting thing is that mathematics is a feature of consciousness, and mathematics has a particularly interesting feature: it does not happen in time. For instance, when expressed physically in a computer program the statement "a = b + 1" takes a finite amount of time to execute, but in mathematics itself "a = b + 1" is instantaneous - zero time to execute, infinite processing speed.

Of course we are capable of doing mathematics, but we don't know if we can do it in infinite speed. Certainly expressing the result of mental computation takes time, but that is to be expected. The real problem is, exactly where is "a = b + 1" instantaneous? From a platonist perspective the answer would be "in abstract reality". If we can mathematically prove that platonism is the best description of reality (and the success of physics does seem to point that way), then we would have basically come up with a proof that the mind is not physical.

Well, as I said philosophers are sloppy, and amateur ones much more so. I just wanted to point out that the solution to the problem we are discussing may be best achieved by mathematics rather than science.

I'd be interested in your thoughts.
 
  • #149
Dichter said:
If the mind is non-physical, I think it's not the scientists who will prove that but the mathematicians. The other alternatives are philosophers and mystics, but the former are too sloppy in their definitions and the latter are not trustworthy.
I agree completely! Although in fairness we should probably consider mathematicians to be scientists also. After all, many if not most modern scientific breakthroughs, at least in physics, are achieved via mathematics as opposed to observation.
Dichter said:
Here is how I see the non-physical nature of the mind could be approached from a mathematical perspective:

We'd have to start with a clear mathematical definition of "physical".
Again, I agree completely!
Dichter said:
I'm not a mathematician, but as far as I know such a definition does not yet exist.
I'm not a mathematician either, but I am aware of an attempt to rigorously define 'reality'. For many people, reality = physical so this definition might suffice for those people. The definition is that 'reality' is defined to be an arbitrary set of numbers. This definition would work equally well for 'physical' and the same conclusions could be drawn. At this point, I hesitate to go further and describe those conclusions for reasons that will become clear at the end of this post. I would like for people to continue reading.
Dichter said:
I don't think it should be difficult though; we do not necessarily have to know exactly what "physical" is, all we have to do is come up with a definition that is consistent with everything we know.
Yes! I agree completely, although the degree of difficulty is relative.
Dichter said:
I would venture that a mathematical definition of "physical" would imply that nothing physical can have infinite or zero magnitude.
I agree. But...the guy I will tell you about later who defined 'reality' says that this is not a necessary condition and it isn't necessarily implied by the definition. (I disagree with him and most mathematicians on the fundamental issue here, but that is another story we could get into, or just check out https://www.physicsforums.com/showthread.php?t=49732)
Dichter said:
Then the next step would be to assert that one or more features of consciousness cannot be physical by that definition.
That assertion would have to be proved. I don't think it is obviously implicit in the definition. Nevertheless, Roger Penrose attempted to prove essentially the same assertion in his "The Emperor's New Mind". In my amateur opinion, he made a convincing case.
Dichter said:
Now the interesting thing is that mathematics is a feature of consciousness
An understatement! I think more attention should be paid to this statement by philosophers, logicians, mathematicians, and biologists alike. I think progress could be made simply by examining the implications of this one statement.
Dichter said:
and mathematics has a particularly interesting feature: it does not happen in time.
Are you sure? How exactly does mathematics "happen"? Here again, I think the examination of this question could open up some interesting possibilities.
Dichter said:
The real problem is, exactly where is "a = b + 1" instantaneous? From a platonist perspective the answer would be "in abstract reality". If we can mathematically prove that platonism is the best description of reality (and the success of physics does seem to point that way), then we would have basically come up with a proof that the mind is not physical.
I agree with the gist of this approach and what it might demonstrate, but as I'm sure you would agree, it is a little loose.
Dichter said:
Well, as I said philosophers are sloppy, and amateur ones much more so. I just wanted to point out that the solution to the problem we are discussing may be best achieved by mathematics rather than science.
I agree completely.
Dichter said:
I'd be interested in your thoughts.
I'm so glad you asked. Readers, brace yourselves.

First (don't worry; this is not so controversial), someone at PF introduced me to George Spencer-Brown's attempt to do essentially what you are suggesting. He wrote "The Laws of Form" which develop a mathematical model of how the universe could have developed from nothing. I haven't learned enough about his work to comment any further than this.

Now, for the controversial part. The guy I mentioned who developed rigorous conclusions from the definition of reality being an arbitrary set of numbers is Dick Stafford. He regularly posts here and is just as regularly misunderstood. My apologies, Dick, but the way you come across in your communication here, as is so frequently and eloquently expressed by Les and others, is completely inconsistent with your actual personality. Being a personal friend of his and having spend many hundreds of hours under his tutelage trying to understand his paper (http://home.jam.rr.com/dicksfiles/reality/Contents.htm ) I can speak with some authority. He is a warm, caring, nice, very smart, but very eccentric guy.

Now, about his paper. With my background being what it is, I was able with great difficulty to follow the development in his Chapter 1 to the point I am convinced that he has discovered a new theorem of mathematics. The theorem roughly says that consistency implies the laws of physics. That is, if you have an arbitrary set of numbers, then there are constraints on the possibilities for specific combinations of subsets of this set. Dick expressed those constraints in a differential equation which I consider to be a theorem. The equation is derived in Chapter 1 strictly from the axioms of analysis. The interesting thing, though, which is presented in his next four chapters, and which I am not competent to follow in detail, is that the known laws of physics are all solutions to Dick's fundamental equation.

The way I see this, in the context of your proposal Dichter, is that Dick has discovered a mathematical way to prove that the behavior of any universe which can be described as a set of numbers can be modeled by his differential equation and therefore must obey the laws of physics. This means that the universe, or at least its phenomenal behavior, could have arisen by something producing a set of numbers. Since numbers are mental concepts, that "something" must be a conscious mind, just as Gregory Bateson suspected. And if we posit that a mental capability was primordial, all that is required is that it developed and increased in complexity and knowledge to the point where it could imagine complex sets of numbers and apply algorithms to operate on those sets. This is what you referred to as mathematics "happening". It seems to me, to answer one of your earlier questions, that mathematics (really computation) can "happen" in one of two ways: either in a conscious mind, or in some machine designed specifically to process algorithms. In the actual production of our physical universe, I suspect that a combination of both types are at work. But now I'm idly speculating again.

Those are my thoughts.

Paul
 
Last edited by a moderator:
  • #150
Paul Martin said:
With my background being what it is, …
Now didn’t you get a masters in mathematics some forty years ago? :smile:
Paul Martin said:
I am convinced that he has discovered a new theorem of mathematics.
Paul, with that theorem idea of yours in mind, I composed a presentation which I last updated a short time ago. If you would look at that and tell me what is difficult to understand, we might be able to create something clear. You can find it at http://home.jam.rr.com/dicksfiles/Explain/Explain.htm . :smile:
Paul Martin said:
The theorem roughly says that consistency implies the laws of physics.
I personally wouldn’t put it that way. What it says is that the basic fundamental elements of any consistent explanation of anything obey the laws of physics. Put another way, that implies any consistent explanation of anything must be built as a consequence of the behavior of fundamental physical entities. In essence I proved that the answer to the question “Can Everything be Reduced to Pure Physics?” is definitively “yes”. :approve:
Paul Martin said:
Since numbers are mental concepts, that "something" must be a conscious mind, just as Gregory Bateson suspected. And if we posit that a mental capability was primordial, … that a combination of both types are at work.
Why not just start with the simple fact that you are aware you can think and call it a Great Original Dilemma. That perhaps the conscious aspect (what one might call the soul) is the primary element of reality (that Great Original Dilemma) and that what everyone calls real is a fabrication of the soul designed to explain the experiences of the soul? Paul, does that sort of fit your picture of the situation? Grapple with that for a while and give me your conclusions. :rolleyes:

Have fun -- Dick

"The simplest and most necessary truths are the very last to be believed."
by Anonymous
 
Last edited by a moderator:
  • #151
Doctordick said:
Now didn’t you get a masters in mathematics some forty years ago?
Yes, but as you are well aware, many of the skills have atrophied since then.
Doctordick said:
Paul, with that theorem idea of yours in mind, I composed a presentation which I last updated a short time ago. If you would look at that and tell me what is difficult to understand, we might be able to create something clear.
Yes, I still owe you that. Be patient.
Doctordick said:
I personally wouldn’t put it that way.
Your way is better. I had the choice of being vague or wrong. I chose vague. When I said, "consistency implies the laws of physics", I conveniently and deliberately didn't specify what it is that must be consistent. You cleared this up by saying that it is the explanations which must be consistent. This relates to a recent comment by Dichter in another thread (https://www.physicsforums.com/showthread.php?p=968280#post968280 )that points out that truth has no meaning except in a linguistic context. That is, the only things that can be true or false are statements in some language. The same is true for consistency. Langauge statements can be consistent or inconsistent, but physical things can't be, unless you choose to call something like quantum superposition inconsistent.

Thank you for providing an improved way of stating your theorem (as I hoped you would).
Doctordick said:
Why not just start with the simple fact that you are aware you can think and call it a Great Original Dilemma. That perhaps the conscious aspect (what one might call the soul) is the primary element of reality (that Great Original Dilemma) and that what everyone calls real is a fabrication of the soul designed to explain the experiences of the soul? Paul, does that sort of fit your picture of the situation? Grapple with that for a while and give me your conclusions.
I did and it was an easy "grapple". That is exactly the approach I have made in coming up with my personal philosophy except the words I use are slightly different.

Instead of "the simple fact that you are aware you can think" I call it "the ability to know". It's exactly the same thing. Mine has the advantage, though, of not using the term 'you', which you use without definition. If we agree that 'you' is a synonym for 'the ability to know', then we are consistent in our language usage, and we agree.

Instead of calling 'the conscious aspect', or the 'soul', by the name 'Great Original Dilemma', with its provocative acronym, I have chosen to use the term 'Primordial Consciousness', or just 'PC'. But in my view, all these terms are synonyms and again our views seem to be consistent.

Instead of referring to physical reality as "a fabrication of the soul designed to explain the experiences of the soul" I have paraphrased Berkeley by saying that physical reality is nothing but ideas in the mind of PC. Here again, except for the words, I think our concepts are completely consistent.

So, yes, that not only sort of fits my picture of the situation, it describes it almost exactly. It seems as if we are even describing the same portion of the elephant, or as you promised me long ago, I think we have a glimpse of the entire elephant.

Thanks for your comments, Dick. I am definitely having fun now.

Paul
 
Last edited:
  • #152
Paul Martin said:
. . . Dick has discovered a mathematical way to prove that the behavior of any universe which can be described as a set of numbers can be modeled by his differential equation and therefore must obey the laws of physics. This means that the universe, or at least its phenomenal behavior, could have arisen by something producing a set of numbers. . . .

. . . Instead of "the simple fact that you are aware you can think" I call it "the ability to know". It's exactly the same thing.

You guys seem to agree that truth can be had by reason alone, but that is disputed by the fact that philosophers had it that way for centuries and produced virtually no knowledge. As William James so simply put it, “To know is one thing, and to know for certain that we know is another.”

Because you can make sense logically or mathematically doesn't mean anything more than the logic and math have been applied according to their own internal rules; it doesn't mean that reality conforms to your perfect logic or math. So what you in an earlier post call "proof" can hardly be that unless you are willing to stipulate that all you mean is your math has obeyed all math rules. It proves absolutely nothing about reality.

Rationalism should be dead and buried by now, but unfortunately its ghost lingers on. The first real progress in philosophy since the Greeks was the empiricists IMO, and that was due to recognizing that making sense doesn't necessarily mean what's been reason will conform to reality. The only way to know if what we've reasoned corresponds to reality is to set up situation where we can observe what's been hypothesized. As Locke said in his An Essay Concerning Human Understanding, “Whence has it all the materials of reason and knowledge? To this I answer, in one word, from experience. In that all our knowledge is founded, and from that it ultimately derives itself.”

If you were to say that all you and Dick are doing is theoretical modeling, then I would be fine with that. It can be fun to follow a model and see how it pans out. But you used the word "proof," and in discussions with Dick in the past he clearly indicated he thinks he "knows" by reason alone, so it seems now you are agreeing with him (if I've misinterpreted your meaning, I apologize). Right there the principles of science have been violated. For science, there is no proof without observation, pure and simple, so I don't know why you or anyone with a Ph.D in science would think otherwise.
 
  • #153
Hi Les,

Thank you for your comments. I am always impressed by what you write. I am dismayed, however, at the difficulty Dick and I have in expressing our views. I really don't think that the three of us are all that far off in our world views. The big differences are in our respective unique areas of expertise, not that I have one, but you guys both do. I think our only real disagreements are semantic.
Les Sleeth said:
You guys seem to agree that truth can be had by reason alone
If it seems that way, then there has definitely been a misunderstanding. I don't believe that much, if any, truth can be had by any means. The only statement I accept as truue is the statement, "Thought happens". Everything else is subject to interpretation and doubt. But there is a notion of mathematical truth, which should more rightly be called consistency. A "true" statement in mathematics is simply one that has been logically demonstrated to be consistent with the accepted body of other mathematical statements. I don't believe that any statement about reality can be proved to be true unless the statement is a tautology, in which case it is merely a statement about labeling and it doesn't say anything about reality.
Les Sleeth said:
So what you in an earlier post call "proof" can hardly be that unless you are willing to stipulate that all you mean is your math has obeyed all math rules. It proves absolutely nothing about reality.
I agree completely. That is all I mean. I insist on the stipulation you mention.
Les Sleeth said:
The first real progress in philosophy since the Greeks was the empiricists IMO, and that was due to recognizing that making sense doesn't necessarily mean what's been reason will conform to reality.
I am less of a philosopher than you so I will accept what you say here. From a mathematical viewpoint, however, the first real progress since Euclid was the recognition that there is no distinction between axioms and postulates. That is, there is no "axiom" that is self-evidently true. They are all arbitrary and have nothing to do with reality. I think that recognition is equivalent to the progress of the empiricists, so I think we agree here.
Les Sleeth said:
The only way to know if what we've reasoned corresponds to reality is to set up situation where we can observe what's been hypothesized.
I agree completely. And, I would add that if we did set up such a situation and observed that nature conforms to our hypothesis, we still can't say that what we've reasoned is true. Instead, all we can say is what you said: we can say what we've reasoned corresponds to reality. Mathematically, this means that the best we can get is an isomorphism between the model and reality. Or, linguistically, all we have is a metaphor that says that reality is something like our model. In any case, all we really have is some agreement to accept some language statements, which we might have the arrogance to call "laws", or "truth". But we really can't say that we know anything about reality.

Nonetheless, we shouldn't downplay those language statements too much. After all, they allow us to function as human beings much more effectively. There is a poem by the mathematician C.R. Wylie, Jr. that I like in one of my college textbooks:

"PARADOX

Not truth, nor certainty. These I foreswore
In my novitiate, as young men called
To holy orders must abjure the world.
'If. . . . , then . . . ,' this only I assert;
And my successes are but pretty chains
Linking twin doubts, for it is vain to ask
If what I postulate be justified,
Or what I prove possesses the stamp of fact.

"Yet bridges stand, and men no longer crawl
In two dimensions. And such triumphs stem
In no small measure from the power this game,
Played with the thrice-attenuated shades
Of things, has over their originals.
How frail the wand, but how profound the spell!"
("Advanced Calculus", by Louis Brand, 1955, Wiley, New York, p. xiv)

It is the "Yet bridges stand..." part that I think we should not overlook. Even though we can't say anything about ultimate reality, we can sure come to a useful understanding of phenomena.
Les Sleeth said:
If you were to say that all you and Dick are doing is theoretical modeling, then I would be fine with that.
Speaking for myself, and I think for Dick as well, that is all I claim we are doing. I think we have no substantial disagreement.
Les Sleeth said:
It can be fun to follow a model and see how it pans out.
I agree. And for those who come up with something really useful, like F=ma, it can also profoundly change human society.
Les Sleeth said:
But you used the word "proof," and in discussions with Dick in the past he clearly indicated he thinks he "knows" by reason alone, so it seems now you are agreeing with him (if I've misinterpreted your meaning, I apologize).
Since I typically don't use the word "proof", I think you are referring to Dick's statement, "In essence I proved that the answer to the question “Can Everything be Reduced to Pure Physics?” is definitively “yes”. " I'll give you my opinion of what he meant. He claims to have proved that the answer "yes" is consistent with the question in the strict mathematical sense. Nothing more. Physics is ultimately a set of language statements that give a method of predicting the outcome of physical experiments. Dick's assertion is that he can demonstrate a mathematical proof, in the sense of Euclid, that all statements that provide a successful methodology for predicting physical phenomena can be derived from his differential equation, which I say is a theorem. Yes, it is an outlandish claim, but as yet no one has shown it to be false.

Now, what Dick "knows" by reason alone, is that his differential equation is a tautology. That is, it is simply another way of stating the axioms of mathematics and their implications.
Les Sleeth said:
Right there the principles of science have been violated. For science, there is no proof without observation, pure and simple, so I don't know why you or anyone with a Ph.D in science would think otherwise.
I don't think otherwise, and I don't think Dick does either. That kind of proof is different from the mathematical proof we have been talking about. And scientists themselves rarely if ever claim that they have proved anything. Instead they only claim to have produced evidence that tends to support an hypothesis.

Now, having said all that, I am really not happy with my response. The problem is that language is just not suitable to express what I want to say. And I think that goes for all of us, particularly for Dick (no offense Dick, but you know that to be "true"). Let me try to express myself another way.

I think the three of us are a microcosm of humanity and that the difficulties we are having in communication are typical. I think the "blind man and the elephant" parable really describes our situation. We are looking at the same thing, we see different parts of it, and we have a hard time expressing to those who haven't seen our part what we have seen. You know you can't adequately describe to us what you experience in meditation -- unless we become competent at it; Dick knows that he can't adequately describe to us what he has discovered mathematically -- unless we become competent at it; I know that I can't adequately describe the goofy crackpot ideas I have come up with without using up so many words that my readers fall asleep. I think both of you have serious and important information that could help extend our understanding of reality. I think that both of your ideas, as best as I can understand them, support and help make sense of my goofy crackpot ideas. This may send a shudder down your spine, but really, I think we are all helping each other see the whole elephant.

Now, I may be getting too close to the deep end, so I'll stop for dinner.

It's good talking to you, Les.

Paul
 
  • #154
Doctordick said:
...Why not just start with the simple fact that you are aware you can think and call it a Great Original Dilemma. That perhaps the conscious aspect (what one might call the soul) is the primary element of reality (that Great Original Dilemma) and that what everyone calls real is a fabrication of the soul designed to explain the experiences of the soul?
Are we starting with the simple fact of "you" or "aware" in this argument of yours ? :confused:
 
  • #155
Les Sleeth said:
You guys seem to agree that truth can be had by reason alone, but that is disputed by the fact that philosophers had it that way for centuries and produced virtually no knowledge.
And that is exactly the reason no one will take the trouble to look down the rabbit hole I have discovered. It is all logic and has to be examined very carefully; something few people are willing to do, particularly if they think the conclusions are ridiculus. :smile: :smile: :smile: :smile:
Paul Martin said:
'If. . . . , then . . . ,' this only I assert;
And my successes are but pretty chains
Linking twin doubts, for it is vain to ask
If what I postulate be justified,
Or what I prove possesses the stamp of fact.
... by Louis Brand
Is an exact statement of what I have done. The proof is a true statement about reality only if the ellipses following that “If” constitute a true statement about reality. If indeed that is the case, then the statement represented by the ellipses following the “then” will also constitute a true statement about reality. And, as he says, the “successes are but pretty chains Linking twin doubts, for it is vain to ask if what I postulate be justified”. The critical issue in my “proof” is that the first set of ellipses stand for the set C, the information upon which the explanation is based. I go to great pains to insure that no constraints whatsoever are placed upon that set: i.e., C can be absolutely anything. This results in the astounding statement: “If (what is being explained is anything) then (an interpretation of the explanation exists which must be a solution to my equation). :rolleyes:

It follows that, if it is true that you are trying to present an internally consistent explanation of something, then it is true that there exists an interpretation of your explanation which satisfies my equation. That makes that equation a pretty fundamental statement. And it is a constraint on your explanation, not a constraint on reality. That is a very profound result with deep and far reaching consequences. :wink:
Les Sleeth said:
As William James so simply put it, “To know is one thing, and to know for certain that we know is another.”
Yes it very much is! :smile:
Les Sleeth said:
It proves absolutely nothing about reality.
I agree with you absolutely! And, since all of modern physics can be deduced from my equation, it points out that modern physics proves absolutely nothing about reality. What I have proved is that “Physics” is essentially a bookkeeping system to keep track of the statistical nature of your experiences and provide a prediction of the future consistent with those experiences. Actually, a rather simple proposition.
Les Sleeth said:
If you were to say that all you and Dick are doing is theoretical modeling, then I would be fine with that.
That is what everyone thinks because they believe that the fact that the philosophers who approached from that direction for centuries failed consistently is a proof that it cannot be done. For how many centuries did educated scholars believe that man could not fly? :smile:
Les Sleeth said:
It can be fun to follow a model and see how it pans out.
Apparently everyone who moves to take me seriously begins by trying to follow the model but, in doing so, they miss the entire import. My model is almost entirely abstract. It is a model of any explanation, not a model of what the explanation explains.
Les Sleeth said:
For science, there is no proof without observation, pure and simple, so I don't know why you or anyone with a Ph.D in science would think otherwise.
Now there you are totally wrong. Science is full of proofs of the kind, “If … then …”. Without understanding logic, a career in science would be unthinkable.
Rade said:
Are we starting with the simple fact of "you" or "aware" in this argument of yours ? :confused:
I feel English is too vague a means of communicating to consider that an issue. It’s all in your interpretation. :biggrin:

Have fun -- Dick
 
  • #156
Paul Martin said:
I don't think otherwise, and I don't think Dick does either. That kind of proof is different from the mathematical proof we have been talking about. And scientists themselves rarely if ever claim that they have proved anything. Instead they only claim to have produced evidence that tends to support an hypothesis.
Doctordick said:
The proof is a true statement about reality only if the ellipses following that “If” constitute a true statement about reality. If indeed that is the case, then the statement represented by the ellipses following the “then” will also constitute a true statement about reality. And, as he says, the “successes are but pretty chains Linking twin doubts, for it is vain to ask if what I postulate be justified”.
I like your answer Paul, but Doctordick seems ambiguous. Is it strictly a mathemathmatical proof without claims that reality must conform? In his response I quoted it seems he believes reality necessarily must conform to pure logic if one's "then" statement is correctly linked logically to an observed "if" statement. That is rationalism, not science.

Because logic and math are used in science doesn't mean they can ever be allowed to constitute the whole of a scientific proof. So I still am of the opinion that while you seem clear about the difference between a scientific and math/logic proof, Doctordick doesn't seem to want to make a clear distinction between the two.
 
Last edited:
  • #157
Les Sleeth said:
Doctordick seems ambiguous
Yes, it's a common problem of his (as it is for most of the rest of us).
Les Sleeth said:
Is it strictly a mathemathmatical proof without claims that reality must conform?
Yes. In my struggle to understand what Dick has discovered, he continually admonished me any time I would infer anything about reality from his work. What has slowly become clear to me after all these years is that his result makes claims only about the relationship between explanations and reality. No claims are made about reality itself. The claim is that if a particular consistent explanation happens to conform to reality (incidentally, we have no way of knowing whether or not it does), then there exists an interpretation of that explanation which satisfies his equation and thus obeys the laws of physics.

I think the ambiguity stems from the two different connotations of 'proof' which we have discussed. I think the three of us are very close to an understanding on this issue.
Les Sleeth said:
In his response I quoted it seems he believes reality necessarily must conform to pure logic if one's "then" statement is correctly linked logically to an observed "if" statement. That is rationalism, not science.
I think you have nailed the difficulty here. If, by 'rationalism' you mean 'mathematics', then yes, Dicks development is rationalism and not science. But just as the Pythagorean Theorem is rationalism (mathematics) it nonetheless is useful in developing the predictive theories of science. Whether or not reality conforms to the Euclidean metric, which is the necessary and sufficient condition for the Pythagorean Theorem to hold, is still open to debate among scientists.

Your statement above, about what seems to be Dick's belief, needs some careful parsing. As I pointed out, it is not reality that must conform; it is that a consistent explanation must conform. And, the conformance is not to "pure logic" but in some sort of analogic or isomorphic way there must be a conformance between the explanation and something real, about which we know nothing.

I think the difference, between Dick's intent and your interpretation of it, is in the things that are linked by the "if" and the "then". In science, both of those things are observations. In Dick's theorem, neither of them is.

Instead, in Dick's Theorem the "if" is a statement about the consistency of some explanation, and the "then" is the assertion that a particular way of interpreting that explanation must exist, viz. one which obeys the laws of physics. No claim of anything about reality is express, and it is only implied if the "explanation" is claimed to have something to do with reality, and such claim does not bear on Dick's result.
Les Sleeth said:
Because logic and math are used in science doesn't mean they can ever be allowed to constitute the whole of a scientific proof.
I agree completely! And I'm sure Dick does too. The whole idea of a scientific "proof" is in the observation of a physical demonstration. Dick's result is pure mathematics (which is why I insist on calling it a theorem) and it applies to science in the same way as any other theorem. None constitutes "the whole of a scientific proof".
Les Sleeth said:
So I still am of the opinion that while you seem clear about the difference between a scientific and math/logic proof, Doctordick doesn't seem to want to make a clear distinction between the two.
I think he does. But if not, I'll let him speak for himself.

This is a very happy day for me, Les, and I have you to thank. I have been dismayed for years at how Dick's development has been misunderstood on these forums and at the difficulty he has had in clearing up the confusion. My work with him has been clouded with the same difficulty, although it has been easier to deal with since there are only the two of us involved, and we have had the opportunity to sit down and talk face to face.

In spite of this advantage, Dick has always maintained that I still don't get it. We have worked specifically on trying to get me to understand what I was missing and we failed. I have a proposed additional section of my website intended to be devoted to a plain English exposition of Dick's result that would finally make clear what it claims, and as yet it is unfinished and unsatisfactory.

But in your recent conversations with Dick, which at times had me wincing, and in this present exchange, it seems to me that it has finally dawned on me what Dick's Theorem actually says. Without these conversations, I may not ever have gotten it. I may be jumping the gun here, and Dick might very well say that I still don't get it, but if so, I still think I have drawn closer. We'll see.

In any case, thank you for your insight, energy, articulation, and persistence.

Paul
 
  • #158
Rade said:
Are we starting with the simple fact of "you" or "aware" in this argument of yours ?
I am very interested in knowing what you see as the difference between "you" and "aware". Are you something different than the awareness that seems to be riding around in your body? If so, what exactly is the difference?

Paul
 
  • #159
Les Sleeth said:
You guys seem to agree that truth can be had by reason alone, but that is disputed by the fact that philosophers had it that way for centuries and produced virtually no knowledge.

That sounds like a strange argument to me. Since when professional incompetence is evidence that a job cannot be done?

If you want to gain knowledge by reason alone, forget philosophers, call in the mathematicians. Sure, mathematicians don't talk about juicy stuff, but at least they make a lot of sense and the truths they have found seem indisputable.

As William James so simply put it, “To know is one thing, and to know for certain that we know is another.”

I wonder if William James had any doubts about two plus two being four.

Because you can make sense logically or mathematically doesn't mean anything more than the logic and math have been applied according to their own internal rules; it doesn't mean that reality conforms to your perfect logic or math.

Reality can perfectly conform to logic if you allow yourself the freedom to conceive of things that are supposed to exist but cannot be directly perceived or measured (things like fields and forces, for instance). Since those things are products of our imagination, they can do whatever it takes to make reality conform to our logical explanations of it. (did I just read a post where someone mentioned phlogistons?)

On the other hand, if you don't have the freedom to give real existence to products of our imagination, then I suspect even "reality" would vanish. As far as I can tell, the notion of a reality underlying our experiences is just a product of our imagination, like fields and forces. And the same can be said for consciousness. I think the only thing we know exists for sure is experience; the existence of anything causing or being caused by experience seems to lack any solid evidence.

(I take it this is equivalent to Paul's "thought happens" being the only absolute truth)

For science, there is no proof without observation, pure and simple, so I don't know why you or anyone with a Ph.D in science would think otherwise.

So when was the last time someone observed gravity? I have only seen things falling, I have never seen something causing things to fall.

It does seem to me people with Ph.D in science do believe in proofs without observation.
 
  • #160
Paul Martin said:
If, by 'rationalism' you mean 'mathematics', then yes, Dicks development is rationalism and not science.

I mean any sort of reasoning about reality without actually attempting to observe what has been reasoned; it can be mathematical reasoning or logical reasoning.


Paul Martin said:
But just as the Pythagorean Theorem is rationalism (mathematics) it nonetheless is useful in developing the predictive theories of science.

Of course! I have no doubts about the usefulness of rational processes in helping us discover the nature of reality. In my own view, I see my reason helping me know where to look for confirming experience, or for experiences that give me the knowledge I am seeking. It's just that I never believe I "know" until I have experienced it.

It seems from Godel's first incompleteness theorem that the math side of things has come to the same conclusion. Consider this explanation from the Wikipedia:

"For any consistent formal theory that proves basic arithmetical truths, it is possible to construct an arithmetical statement that is true 1 but not provable in the theory. That is, any consistent theory of a certain expressive strength is incomplete.

Here, "theory" refers to a set of statements. (A theory is in general an infinitely large set.) A theory is "consistent" if it does not prove any contradictions. The meaning of "it is possible to construct" is that there is some mechanical procedure which when given the axioms of the theory, produces another statement. That this statement is not provable in the theory means that it cannot be derived from statements of the theory using the standard rules of first-order logic. The statement produced by the procedure is often referred to as "the Gödel sentence" for that theory, though there are actually infinitely many statements that have the same property (of being true but not provable from the theory)."



My non-mathematical interpretation of that would say any consistant theory contains assumptions which are not proven by the theory itself, and therefore no theory can either self-prove or generate new statements which are proven. In the real world, this theorem seems to correspond to the practice of empiricism where we reason a hypothetical path and then follow it by attempting observation. We can't just reason the path and claim we know anything, yet it has seemed to me that is just what Doctordick has been saying.


Paul Martin said:
. . . the conformance is not to "pure logic" but in some sort of analogic or isomorphic way there must be a conformance between the explanation and something real, about which we know nothing.

The epistomology known as correspondence seems to cover this. I'll grant that the ambiguous use of the term "proof" might be the problem, but that still leaves me wondering about somehting Doctordick seem to imply.

For example, you/he says:


Paul Martin said:
Instead, in Dick's Theorem the "if" is a statement about the consistency of some explanation, and the "then" is the assertion that a particular way of interpreting that explanation must exist, viz. one which obeys the laws of physics. No claim of anything about reality is express, and it is only implied if the "explanation" is claimed to have something to do with reality, and such claim does not bear on Dick's result.

Is this to be interpreted strictly within the context of theoretical physics? If so, then I have nothing more to say since it isn't what I am concerned about in this thread. In this thread we are talking about the "reality" of the mystical experience, and so when you say ". . . that explanation must exist, viz. one which obeys the laws of physics," is Doctordick's idea that to be real an explanation must have a physical counterpart?

I must say that in past discussions, such as in "Can Everything be Reduced to Pure Physics," I have thought he has said exactly that. If that is the case, then he's built in an a priori assumption about the nature of reality that automatically acts a filter. Yes, everything physical can be reduced to pure physics, but that doesn't imply that everything is physical.

In regard to the mystical experience, we may not even be able to fit it to the math side . . . what if it is purely known on the experiential side? If so, the demand that it be revealed by any theoretical process is futile.
 
  • #161
Les Sleeth said:
[By 'rationalism'] I mean any sort of reasoning about reality without actually attempting to observe what has been reasoned; it can be mathematical reasoning or logical reasoning.
Then 'rationalism' is different from 'mathematics' and we need to revisit this point. You say that rationalism is "reasoning about reality". Mathematics is not about reality, but is only about abstract entities that may or may not have anything to do with reality. It is the scientists who make the hypothetical connections between those abstract entities and something they consider to be real. Sometime those connections lead to useful insights and sometimes they don't. But either way, the connections are outside the boundaries of mathematics. With this distinction being made, Dick's work is mathematical and not rationalism.
Les Sleeth said:
My non-mathematical interpretation of that would say any consistant theory contains assumptions which are not proven by the theory itself, and therefore no theory can either self-prove or generate new statements which are proven.
I would agree with your interpretation if you change 'assumptions' to 'propositions'. But I see this interpretation as different from your assertion that, "It's just that I never believe I "know" until I have experienced it." When you say, "know", I think you are talking about something real. In the context of Goedel's Theorem, the only applicable sense of "know" is whether we can be certain that certain propositions are consistent. Goedel's conclusion is that we can't "know" in that sense about some abstract language statements. Your conclusion is that you can't "know" about some aspect of reality without experience. There is a logical parallel, but the two contexts are quite different.
Les Sleeth said:
We can't just reason the path and claim we know anything, yet it has seemed to me that is just what Doctordick has been saying.
Maybe it will help clear things up if we are more specific. We can't just reason the path and claim we know anything about reality. We can, however, reason the path and claim that we know something about the relationships between some language statements. The language statements have nothing to do with reality, and the relationships are strictly defined by arbitrary rules. I think that is what Doctordick has been saying.
Les Sleeth said:
Is this to be interpreted strictly within the context of theoretical physics? If so, then I have nothing more to say since it isn't what I am concerned about in this thread. In this thread we are talking about the "reality" of the mystical experience
No, it isn't to be interpreted strictly within the context of theoretical physics. It applies to any context which has any consistent explanation. Of course physics attempts to come up with consistent explanations for real (physical) phenomena, so it applies there. But, if some consistent explanation could be arrived at for mystical experience, then Dick's result would apply there as well. It would say that if that explanation were indeed consistent, then there would be some way of interpreting it such that the laws of physics would obtain in that interpretation. It would impose a constraint on the possible consistent explanations for mystical experience, which I think would be important to understand when trying to come up with explanations.

In my view, it nets down to two possibilities, which might exist in any combination: parts (or aspects) of reality might admit consistent explanations, and in those cases, reality must conform to the laws of physics. Other parts (or aspects) of reality might not admit any consistent explanation, and in those cases, reality may behave in defiance of the laws of physics. My guess is that there is some of both going on with respect to mystical experiences and that in the physical world, above the threshold of the HUP, all aspects can be consistently explained, and thus the laws of physics are obeyed.
Les Sleeth said:
... when you say ". . . that explanation must exist, viz. one which obeys the laws of physics," is Doctordick's idea that to be real an explanation must have a physical counterpart?
No. First of all, he never talks about an explanation being real. He talks about explanations being consistent or not. Secondly, if any connection is made between an explanation and reality, that is done outside the context of Dick's theorem.
Les Sleeth said:
In regard to the mystical experience, we may not even be able to fit it to the math side . . . what if it is purely known on the experiential side? If so, the demand that it be revealed by any theoretical process is futile.
I am just a little less pessimistic than that. I think there may be some aspects of the mystical experience that would lend themselves to math and logic. If so, some explanations for some of it might be possible. But your guess here is probably much better than mine.

Paul
 
  • #162
Les Sleeth said:
My non-mathematical interpretation of that would say any consistant theory contains assumptions which are not proven by the theory itself,

NO! The theory is derived from a finite set of assumptions (axioms) and syntactic rules and contains STATEMENTS that can be formed according to the rules that are not CONCLUSIONS from the axioms. There is nothing about ADDITIONAL ASSUMPTIONS.

In the real world, this theorem seems to correspond to the practice of empiricism where we reason a hypothetical path and then follow it by attempting observation

The theorem is about axiomatic theories of arithmetic (and has been shown to apply to computer programs too), it has no applicabilitity to the general world. For example geometry does not fall under the theorem.
 
  • #163
Dichter said:
That sounds like a strange argument to me. Since when professional incompetence is evidence that a job cannot be done?

I wouldn’t characterize it as incompetence (out of respect for some great thinkers), but I would say they were missing a crucial piece of the epistemological puzzle. Because they lacked that piece, they tried even harder to make rationalism work, which produced libraries of speculation and little more.


Dichter said:
If you want to gain knowledge by reason alone, forget philosophers, call in the mathematicians. Sure, mathematicians don't talk about juicy stuff, but at least they make a lot of sense and the truths they have found seem indisputable.

Surely you jest! I wonder if you know what we are talking about. We are discussing how one knows, not how one calculates. Do you think someone could sit in his room, mathematically figure out reality, and then actually know he has figured it out without going out and experiencing reality? Calculations tell us if the math is correct, it doesn’t tell us if it corresponds to reality; we only know that if we observe. That observation is the “missing piece” rationalistic philosophers lacked.


Dichter said:
I wonder if William James had any doubts about two plus two being four.

In addition to being a devoted empiricist, James was also a fan of philosophical pragmatism. One thing you can most definitely say about empiricism is that it “works,” evidenced by all the technology it has produced. The meaning of “empirical” is experience, so I am certain James, a brilliant philosopher, knew that the addition of experience to rationality (and rationality of course includes math) was what was making philosophy finally “work.”


Dichter said:
Reality can perfectly conform to logic if you allow yourself the freedom to conceive of things that are supposed to exist but cannot be directly perceived or measured (things like fields and forces, for instance). Since those things are products of our imagination, they can do whatever it takes to make reality conform to our logical explanations of it. (did I just read a post where someone mentioned phlogistons?)

Except it isn’t true that fields and forces can’t be measured or perceived, that is exactly why we know about them. And why do you think after some unknown particle is indicated by math the next step scientists take is to look for it? In terms of research, experience is the foundation of modern epistemology, math and logic serve to predict what to look for.


Dichter said:
On the other hand, if you don't have the freedom to give real existence to products of our imagination, then I suspect even "reality" would vanish. As far as I can tell, the notion of a reality underlying our experiences is just a product of our imagination, like fields and forces. And the same can be said for consciousness.

I am not sure I get your point. We are consciousness, and so we that’s all we have to work with. I don’t see the value in doubting its ability to accurately perceive reality.


Dichter said:
I think the only thing we know exists for sure is experience . . .

I like this thought, but you seem to vacillate about how much you believe it.


Dichter said:
. . . the existence of anything causing or being caused by experience seems to lack any solid evidence.

Well, what causes it lacks conclusive evidence, but I know for a fact that my experience causes me to know I exist.


Dichter said:
(I take it this is equivalent to Paul's "thought happens" being the only absolute truth)

Sounds like rationalism.


Dichter said:
So when was the last time someone observed gravity? I have only seen things falling, I have never seen something causing things to fall.

To “observe” isn’t necessarily limited to our visual sense; to hear is to observe and to feel is to observe. Gravity can definitely be felt. But even if it is true that gravity is only inferred from its effects, it isn’t a big deal if you can see it working and working consistently. No matter where we’ve gone in the universe so far the effects of gravity (and acceleration) have held. (Of course, nobody really understands the “why” of gravity.)


Dichter said:
It does seem to me people with Ph.D in science do believe in proofs without observation.

IMO, only the foolhardy believe something is proven about reality without observation. How do you think philosophy got the reputation of mental masturbation? After all the centuries of trying to know without the observation requirement, and seeing the vast amount of baloney written (albeit clever baloney), nobody with half an education should still be trying to understand reality only intellectually.
 
  • #164
selfAdjoint said:
NO! The theory is derived from a finite set of assumptions (axioms) and syntactic rules and contains STATEMENTS that can be formed according to the rules that are not CONCLUSIONS from the axioms. There is nothing about ADDITIONAL ASSUMPTIONS..

Thank you, that's what I meant.
 
  • #165
Paul Martin said:
Then 'rationalism' is different from 'mathematics' and we need to revisit this point. You say that rationalism is "reasoning about reality". Mathematics is not about reality, but is only about abstract entities that may or may not have anything to do with reality.

I'd say math is rationalistic and obviously a most useful tool. Rationalism as I mean it is the improper inference from reason or math that some never-observed aspect of reality necessarily follows.


Paul Martin said:
I would agree with your interpretation if you change 'assumptions' to 'propositions'. But I see this interpretation as different from your assertion that, "It's just that I never believe I "know" until I have experienced it." When you say, "know", I think you are talking about something real. In the context of Goedel's Theorem, the only applicable sense of "know" is whether we can be certain that certain propositions are consistent. Goedel's conclusion is that we can't "know" in that sense about some abstract language statements. Your conclusion is that you can't "know" about some aspect of reality without experience. There is a logical parallel, but the two contexts are quite different.

Yes, sorry . . . I rushed through that little point and slipped up. Also, I probably got too creative in my attempt to draw a clear distinction between reason and experience.

I don't have time to answer your entire post right now, but I like the discussion.

Let me say however that your representation of Doctordick's beliefs are not what I've experienced with him. I don't think I have ever read him where I didn't sense he thought one could know some of the major things by reason alone (and often that if who he was addressing would just put themselves under his tutelage he would show them how to know reality through his methods).
 
  • #166
Les Sleeth said:
We are discussing how one knows, not how one calculates. Do you think someone could sit in his room, mathematically figure out reality, and then actually know he has figured it out without going out and experiencing reality? Calculations tell us if the math is correct, it doesn’t tell us if it corresponds to reality; we only know that if we observe.

Well, it's not what I would call likely, but it is logically possible. A theory could have the properties that (a)It is unique, and (b) It is necessary, and (c) It accounts for all the thinker's experiences. Then it would be as valid as, say any theory based on personal experience of unity.
 
  • #167
Paul Martin said:
I am very interested in knowing what you see as the difference between "you" and "aware". Are you something different than the awareness that seems to be riding around in your body? If so, what exactly is the difference? Paul
Thank you for the question. The difference as I see it is one of constraint. There is only one "you", but a range of possibilities of being "aware", including not being aware. Perhaps it is an axiom that all living entities are constrained in awareness but not in being.
 
  • #168
selfAdjoint said:
Well, it's not what I would call likely, but it is logically possible. A theory could have the properties that (a)It is unique, and (b) It is necessary, and (c) It accounts for all the thinker's experiences. Then it would be as valid as, say any theory based on personal experience of unity.

I agree it is possible to mathematically compose a theory as valid as one based on personal experience, or even to come up with one that surpasses. But that wasn't my point.

What I was trying to say was, whether or not one has got a theory right cannot be known unless one is able to experience what one has hypothesized. You can "believe" you can have "faith" you can be supremely logical . . . but none of it gives one knowledge. Only experience delivers that.

So you might have hit the nail exactly on the head, but you will never know it unless you can confirm it by experiencing the reality you have predicted.
 
  • #169
Hi Les, we seem to verging on communication here. I think Paul’s presence is a valuable asset and I hope you don’t think we are ganging up on you. :smile:
Les Sleeth said:
I like your answer Paul, but Doctordick seems ambiguous …
Sorry about that. I think it stems from the fact that I have thought about issues related to my proof for some 50 years and my concept of what is going on here is probably quite alien to both of you. That is not a criticism of either of you, it is just that I suspect that what I am thinking and what you think I am thinking are quite different things. When it comes to thoughts and communications, there are three very important and vastly different concepts which I think we should take care to recognize. :redface:

There are symbols (words, letters, pictures, sounds, gestures, ascII code, Chinese characters, Egyptian hieroglyphs etc.) all are intermediaries in communications and, if they are to be understood, they must be learned. Learning these symbols is exactly the same problem as is learning anything else. It should be recognized that the idea that one understands these symbols can no more be proved than one can prove any other theory. The assumption that the symbols are understood is supported only by the fact that, once we understand them, our interchanges via them make sense to us. That is, we no longer have to make adjustments to our understanding of them as we are no longer surprised by their usage.

There are concepts (mental images together with the connections associated with those ideas). Concepts are totally internal to each of us. They are private in the sense that we can not deliver a concept in our head to another mind. We can name them via symbols and, if we have a sufficiently large collection of concepts already named via symbols and reasonably well understood, we can perhaps communicate the concept we have in mind to another. But it must be recognized that the fact that a concept has been communicated is an assumption. Again, this assumption is supported only by the fact that, once the concept is understood, usage of the symbol we have attached to that concept no longer surprises us in our communications.

Finally, semantics generally includes a referent (real world object). This is supposed to be the actual thing being referred to. However, it must be recognized that it also is assumed. It is an element of our experiences. If a particular experience is repeated often enough, we will mentally develop a concept of that experience. That concept will include relationships with other experiences and, if the concept continues to be consistent with our experiences we will assume it is a referent to a real thing and not an illusion. Once again, the only support is the fact that we are no longer surprised by the experiences associated with that referent. We “know” that experience and can recognize it.

Lack of surprise is the thread through all these components. The existence of reality is an assumption. When I say that I am neither saying reality does not exist (solipsism) nor am I saying that reality does exist (realism), I am merely saying that we don’t know the answer. Our only ally in our search for truth is finding an explanation which yields expectations consistent with our experiences: i.e., a good explanation eliminates surprises. If we have a decent explanation, we will not be surprised by experiences covered by that explanation. o:)

The point of all that being that everything has to be learned and nothing is a-priory known. Thus it is that your interpretation of any phenomena is dependent upon the information you have acquired with regard to that phenomena (where what I mean by the term “phenomena” is a completely open issue). I consider your interpretation to be equivalent to a personal explanation of whatever is being referred to by the term phenomena. :rolleyes:

Now, I have defined an explanation to be a method of obtaining expectations from given known information. If you believe that definition does not include your concept of an explanation, then you need to either show me an example of what you believe to be an explanation which does not provide any information as to your expectations or something which yields your expectations which can not be seen as an explanation. :confused:

Now, if you will concur with me that I have defined a concept consistent with the common comprehension of an explanation then I can guarantee you that there exists a totally consistent interpretation of the symbols used to communicate that explanation which requires the fundamental elements of that explanation to obey my equation if it is an internally consistent explanation. The issue of reality has absolutely nothing to do with that proof. And it is a constraint on your explanation, not a constraint on reality. (Check message #255 on this thread)
Les Sleeth said:
Because logic and math are used in science doesn't mean they can ever be allowed to constitute the whole of a scientific proof. So I still am of the opinion that while you seem clear about the difference between a scientific and math/logic proof, Doctordick doesn't seem to want to make a clear distinction between the two.
Oh, I very much make the distinction, or perhaps attempt to make the distinction; however, having discovered my proof (that any internally consistent explanation of anything must be a solution to my equation) I have yet to find a single possible experiment which would differentiate between “absolutely anything is possible” and “it has to obey the laws of physics”. I have found no instance where any physical experiment failed to be a solution to my equation. It seems to me that I can take that fact as strong evidence that my deduction is equivalent to observation (and that would be the observations of thousands of professional researchers dedicated to careful observation over many many centuries). :redface:
Paul Martin said:
What has slowly become clear to me after all these years is that his result makes claims only about the relationship between explanations and reality.
How about “relationships between explanations and the behavior of the fundamental elements of those self same explanations. If the explainer claims those fundamental elements are part of reality, then and only then can one interpret my results as making claims about the relationship between explanations and reality; however, that makes the assumption that the explainer's explanation is correct. :rolleyes:
Paul Martin said:
No claims are made about reality itself. The claim is that if a particular consistent explanation happens to conform to reality (incidentally, we have no way of knowing whether or not it does), then there exists an interpretation of that explanation which satisfies his equation and thus obeys the laws of physics.
I only have one cavil with this statement Paul. It is not necessary that the explanation conform to reality. If it is an internally consistent explanation, there exists an interpretation of it which satisfies my equation and thus obeys the laws of physics. In fact, it would be nice to find a solution to my equation which does not conform to reality as we know it. We could then design an experiment to investigate that solution. If that solution could not be found represented in reality, we would know that there would have to be a law preventing such a real solution. We would finally know something definitive about reality. :devil:

You are all throwing the term “rationalism” around enough here to get me to look it up. It seems that “rationalism” is an approach to philosophy based on the thesis that human reason can, in principle, be the source of all knowledge. If that is the meaning you all intend, I can assure you that I am not making any attempts whatsoever to conform to the rationalist thesis. In fact, I am doing exactly the opposite. I began my work trying to exactly lay out the proper constraints a scientist should observe in order to avoid wasting his time with experiments where the results of the experiments provide no additional information. That would be where the outcome of the experiment was preordained by his definitions. For example, if down hill is to be defined via a carpenter’s level, an experiment to determine if water runs downhill is an utter waste of time. Since the bubble in the level is the absence of water, downhill has been defined to be the direction the water ran.

When I had finally laid out the logical constraints one should consider prior to designing their experiment I found I could express those constraints in the form of an equation which, over the years, I have come to call my fundamental equation. Initially, the most serious difficulty was that the equation was quite meaningless as I could not solve it. It took me over five years before I found the first solution. Now I fully expected the result to be a constraint, on relations, not a predictor of anything (after all the construct is clearly a tautology). What I discovered was that all the fundamental equations of modern physics were either solutions to that equation or approximate solutions to it. The only interpretation of that result that I can conceive of is that physics itself is a complex tautology: i.e., it is all true by definition. That was over twenty years ago and, to this day, I am aware of no experiment which would violate the rationalist thesis. In my thoughts, there is only one experiment I can conceive of which would provide such a violation. As I said above, that would be to find a solution to my equation which could not be found in reality. I have not yet found one and certainly no one else will find one unless they at least understand it enough to search for such a solution. :cry:
Paul Martin said:
Les Sleeth said:
So I still am of the opinion that while you seem clear about the difference between a scientific and math/logic proof, Doctordick doesn't seem to want to make a clear distinction between the two.
I think he does. But if not, I'll let him speak for himself.
My proof has nothing to do with what you are calling “scientific proof”. When I say it is a proof, I mean a logical proof pure and simple.
Paul Martin said:
Dick has always maintained that I still don't get it. We have worked specifically on trying to get me to understand what I was missing and we failed.
Paul, you are as close to understanding what I am saying as anyone and every time we have added to our conversations I think you have improved. What I am saying is that any internally consistent explanation of anything can be interpreted in a way which requires my equation to be true. And you are perfectly right, that statement has nothing to do with reality as it has only to do with the explanation. The interesting fact, which I think you are getting very close to comprehending, is that, since the behavior of the fundamental elements of that explanation are exactly what has been theorized to be the behavior of the fundamental elements of modern physics, the phenomena of modern physical reality can be seen as the foundation of any explanation of anything. It is that fact which answers the question, “Can everything be reduced to physics?” in the affirmative. :smile:

And finally, to Less Sleeth, I have put forth no theory, I have put forth a logical deduction. It is interesting that you have brought up rationalism. Given what I have shown, if I were to put forth a theory, that theory would be that the rationalist thesis is correct. And, as I have already said twice, the scientific proof that the rationalist thesis is in error would be to find a solution to my equation which describes a phenomena which cannot be found in reality. From my position, I see that as a distinct possibility so I would not yet call myself a rationalist. o:)
Paul Martin said:
But in your recent conversations with Dick, which at times had me wincing, and in this present exchange, it seems to me that it has finally dawned on me what Dick's Theorem actually says. Without these conversations, I may not ever have gotten it. I may be jumping the gun here, and Dick might very well say that I still don't get it, but if so, I still think I have drawn closer. We'll see.
Paul, I think that, if you have missed some point, it is only a subtle nuance of the presentation. I would say that you are so close to understanding exactly what I am saying that the differences could almost be called trivial. :biggrin: :!)

Les Sleeth on the other hand is confusing evidence that the rationalists are correct with a thesis that the rationalists are correct and he is essentially refusing to look at the evidence. Dichter, I appreciate your thoughtful comments though I think you are missing the essence of some of Les’s complaints. I enjoy your forthright comments none the less. :smile:
Les Sleeth said:
So you might have hit the nail exactly on the head, but you will never know it unless you can confirm it by experiencing the reality you have predicted.
At the risk of repeating myself, I have found no instance where any physical experiment failed to be a solution to my equation. It seems to me that I can take that fact as very strong evidence that my deduction appears to be equivalent to observation (and that would be the observations of thousands of professional researchers dedicated to careful observation over many many centuries). :smile:

Have fun -- Dick

"The simplest and most necessary truths are the very last to be believed."
by Anonymous
 
  • #170
Rade said:
There is only one "you"
Well, that depends on perspective. There is only one "me", but from my perspective, there is not only "you" but there are also about six billion other people whom I would also address as "you" if I were to talk to them.
Rade said:
a range of possibilities of being "aware", including not being aware
As I said above, there are six billion possibilities of being "you", as well as the possibility of not being you.

This is, of course a silly game with words. But what I was getting at with my question was your thoughts about what constitutes your essential identity. It seems to me that one's awareness constitutes one's identity and it is that identity that is addressed by the pronoun 'you'.
Rade said:
Perhaps it is an axiom that all living entities are constrained in awareness but not in being.
I think we know of no axiom that is self-evidently true, if that's what you mean by 'axiom'. As for constraints, I would agree that living entities are constrained in awareness. But don't you think they are also constrained in being? After all aren't they constrained in extent, in location, in possibilities, in time, etc. etc.?

Paul
 
  • #171
Doctordick said:
How about “relationships between explanations and the behavior of the fundamental elements of those self same explanations. If the explainer claims those fundamental elements are part of reality, then and only then can one interpret my results as making claims about the relationship between explanations and reality; however, that makes the assumption that the explainer's explanation is correct.
Yes, I'll buy that. That way you don't have to make any claims whatever about reality. I think that we actually can say nothing meaningful about reality at all -- except maybe that statement itself.
Doctordick said:
I only have one cavil with this statement Paul. It is not necessary that the explanation conform to reality. If it is an internally consistent explanation, there exists an interpretation of it which satisfies my equation and thus obeys the laws of physics.
Yes. That should have been obvious to me from the math. The explanation might explain something, or it might explain nothing. But if it is consistent, then your equation must hold. As you have been saying all along, it is much more general than physical reality, or reality at all.
Doctordick said:
What I am saying is that any internally consistent explanation of anything can be interpreted in a way which requires my equation to be true.
You have been saying that for a long time. It is finally making sense to me. Sorry I'm so slow, but as you well know, that's how I am.

Paul
 
  • #172
Doctordick said:
There are concepts (mental images together with the connections associated with those ideas). Concepts are totally internal to each of us. They are private in the sense that we can not deliver a concept in our head to another mind. We can name them via symbols and, if we have a sufficiently large collection of concepts already named via symbols and reasonably well understood, we can perhaps communicate the concept we have in mind to another. But it must be recognized that the fact that a concept has been communicated is an assumption. Again, this assumption is supported only by the fact that, once the concept is understood, usage of the symbol we have attached to that concept no longer surprises us in our communications.

Right, I understand the basis of thought and communicating. I’ve looked ahead at the line of reasoning you will take and I think you are about to assume a couple of things I don’t. Just to give you a preview of where I am headed with this critique, I don’t think you are careful enough in distinguishing between what might be called pure experience, and the processes of reason.

Since I’ve relied on William James already in this thread (though I am not a disciple or anything), let me stick with his model a bit to help explain “pure experience.” Here’s a link to James’ paper, Does Consciousness Exist:

http://fair-use.org/william-james/essays-in-radical-empiricism/does-consciousness-exist

He intends the term “consciousness” to mean rational thought (which is not agreed on today as a suitable definition of consciousness). But his model is useful in that he assumes a fundamental substrate that thinking happens within, what he calls pure experience.

How can he say consciousness (viz. rationality) does not exist? It’s because it is not an entity (endurance as an identity), but rather an ever-changing process, and one which turns instantly invisible as it completes its functions (functionalists here likely would be satisfied consciousness is explained). However something does remain, and that is the more basic substrate of experience which is the knower of some object known.

I’m going to stop using James soon, but I hope you get the fundamental concept that thinking happens within something more basic which is absorbing, understanding, realizing . . . knowing what the process of thought acts on.


Doctordick said:
Finally, semantics generally includes a referent (real world object). This is supposed to be the actual thing being referred to. However, it must be recognized that it also is assumed. It is an element of our experiences. If a particular experience is repeated often enough, we will mentally develop a concept of that experience. That concept will include relationships with other experiences and, if the concept continues to be consistent with our experiences we will assume it is a referent to a real thing and not an illusion. Once again, the only support is the fact that we are no longer surprised by the experiences associated with that referent. We “know” that experience and can recognize it.

Yes, more good characterization of thought processes. But I will argue that we don’t need the intermediary “concept” to know, as you seem to imply. If that were true, how could a dog (as I’ve asked a couple of times already) know his way home? We need concepts to think, but not to know.


Doctordick said:
Lack of surprise is the thread through all these components. The existence of reality is an assumption. When I say that I am neither saying reality does not exist (solipsism) nor am I saying that reality does exist (realism), I am merely saying that we don’t know the answer. Our only ally in our search for truth is finding an explanation which yields expectations consistent with our experiences: i.e., a good explanation eliminates surprises. If we have a decent explanation, we will not be surprised by experiences covered by that explanation.

Well, here’s where we significantly diverge. Our concepts about reality are assumptions. To doubt the existence of realty we’d need reason to doubt the ability of consciousness to perceive existence, and we have no reason to doubt that.

Here’s my last dependence on James, but if you read the paper I referenced he mentions pragmatic tests. Reasoning with a kindred test, why should we doubt that consciousness experiences existence when we work with what exists and it both behaves as we perceive and we are able to manipulate stuff as we’ve perceived? The pragmatic test tells us, for example, that if we perceive cold as hot, then when we apply hot to an ice cube it should surprise us and stay frozen . . . but that’s not what happens. Our perception “works.” I can see an objection coming to that point so let me clarify further that we have to be really careful here with our terms.

Let’s take a bug crawling across sidewalk. He scurries along and then encounters a twig. He stops, feels it with his antennae, and moves around it. Now, I say he “knows” that something in his path exists; but he doesn’t “know” in a way you do.

You have names/symbols for the twig, you have concepts for its makeup, you have theories about its origin, and so on. In other words, you have a conceptual framework the bug doesn’t. So what’s happening is that both you and the bug “know” a twig exists, but only one of you can conceptualize about it. Two completely different things but you are using one word— know—to describe them. Intellectual certainty is one thing, and experiential certainty is another. The intellect requires a specific logic to be satisfied as to the “what,” but experiential certainty only requires an encounter to know something exists.

If I had my way, I would use the term “know” to describe experiential encounters with existence, and I would use the term “accurately explained” to characterize conceptualizing that nicely corresponds to how things are ordered, organized, composed, etc.


Doctordick said:
The point of all that being that everything has to be learned and nothing is a-priory known. Thus it is that your interpretation of any phenomena is dependent upon the information you have acquired with regard to that phenomena (where what I mean by the term “phenomena” is a completely open issue). I consider your interpretation to be equivalent to a personal explanation of whatever is being referred to by the term phenomena.

Exactly, and here you clearly state the situation. There is the experience of existence, and there is the interpretation. Two TOTALLY different things.


Doctordick said:
Now, if you will concur with me that I have defined a concept consistent with the common comprehension of an explanation then I can guarantee you that there exists a totally consistent interpretation of the symbols used to communicate that explanation which requires the fundamental elements of that explanation to obey my equation if it is an internally consistent explanation. The issue of reality has absolutely nothing to do with that proof. And it is a constraint on your explanation, not a constraint on reality. (Check message #255 on this thread)

I agree reality has nothing to do with your proof. Do I need to point out that you are merely confirming what I’ve been saying all along?


Doctordick said:
Oh, I very much make the distinction, or perhaps attempt to make the distinction; however, having discovered my proof (that any internally consistent explanation of anything must be a solution to my equation) I have yet to find a single possible experiment which would differentiate between “absolutely anything is possible” and “it has to obey the laws of physics”. I have found no instance where any physical experiment failed to be a solution to my equation.

I don’t know if you have discovered such an explanation, but I agree one is possible. But take pause here . . . remember, I’ve read many things you have said in the past, and one of them is that EVERYTHING can be reduced to physics, which you’ve just said again.

Now, just because you express something in physical terms doesn’t mean it has been properly expressed, so I assume you accept the requirement that expressions of "everything" are complete. In other words, if love has a physical aspect and a non-physical aspect, then the statement “love can be reduced to physics” means you think you can account for all aspects of love with physics (i.e., and not just that you are only accounting for the physical aspects). It is my recollection that you have said physics can cover it all many times.


Doctordick said:
It seems to me that I can take that fact as strong evidence that my deduction is equivalent to observation (and that would be the observations of thousands of professional researchers dedicated to careful observation over many many centuries).

Do you see my objection yet? There is no rationalistic equivalent to experience! They are two utterly distinct processes. “Knowing” is an experiential product, and “understanding” is a rational product.



Doctordick said:
Les Sleeth on the other hand is confusing evidence that the rationalists are correct with a thesis that the rationalists are correct and he is essentially refusing to look at the evidence.

Oh really? What evidence would that be? You above stated as plainly as one can that “my deduction is equivalent to observation.” That sir, unlike your claims, is unambiguous rationalism.
 
Last edited:
  • #173
Well Les, the only thing I can really say to summarize your post is that, "The simplest and most necessary truths are the very last to be believed." I think you have made some major misinterpretations of what I said, and Paul is probably correct: that it is my fault for failing to be clear. Paul, maybe you can help me out here as I think you pretty well know what I am talking about. :redface:
Les Sleeth said:
But I will argue that we don’t need the intermediary “concept” to know, as you seem to imply.
I don't think I implied any such thing. Perhaps you drew that conclusion from my use of the word "object" in the third semantic reference. I did not intend the word to be constrained to any specific character of thing. With that correction, this is what I meant to say:
Doctordick said:
Finally, semantics generally includes a referent (real world thing). This is supposed to be the actual thing being referred to. However, it must be recognized that it also is assumed. It is an element of our experiences. If a particular experience is repeated often enough, we will mentally develop a concept of that experience. That concept will include relationships with other experiences and, if the concept continues to be consistent with our experiences we will assume it is a referent to a real thing and not an illusion. Once again, the only support is the fact that we are no longer surprised by the experiences associated with that referent. We “know” that experience and can recognize it.
How can you know something you cannot be aware of or refer to? Just what is it that you could know in such a circumstance? :confused:
Les Sleeth said:
If that were true, how could a dog (as I’ve asked a couple of times already) know his way home? We need concepts to think, but not to know.
Now here I think you considerably underestimate the mental powers of a dog. We just got back from Denver visiting our pregnant daughter. (First grandchild after pretty well giving up hope!) Anyway, they have a dog. When they ask the dog to get the paper, he gets up and gets the paper. When they ask the dog if he wants to go out he goes to the door. When they ask the dog if he wants to go for a walk he wags his tail and gets his leash. :smile:

Now are you going to convince me that dog has no concept of getting the paper, going out or going for a walk? Or better than that, that a normal dog has no concept of "food"? If a squirrel has no concept of food, how come he doesn't eat rocks? :confused:
Doctordick said:
To doubt the existence of realty we’d need reason to doubt the ability of consciousness to perceive existence, and we have no reason to doubt that.
To set any part of our perceptions above examination is to scuttle rational science. Considering the almost unbelievable breadth of illusions which the subconscious human mind can create, it rather seems to me that we have fairly good reasons to doubt the veracity of our perceptions on any issue. And I think quite a little support exists for such doubts. :rolleyes:
Doctordick said:
The existence of reality is an assumption. When I say that I am neither saying reality does not exist (solipsism) nor am I saying that reality does exist (realism), I am merely saying that we don’t know the answer.
If we are going to remain objective we must allow for the possibility we could be wrong. You make no such allowance whereas, via the introduction of the set C, I certainly allow for the possibility that you are correct. :wink:
Les Sleeth said:
Now, I say he “knows” that something in his path exists; but he doesn’t “know” in a way you do.
Once again, you have made an assumption without real justification. How do you "know" that the bug's knowledge differs from mine in any manner other than complexity? :confused:
Les Sleeth said:
If I had my way, I would use the term “know” to describe experiential encounters with existence, and I would use the term “accurately explained” to characterize conceptualizing that nicely corresponds to how things are ordered, organized, composed, etc.
Aren't you sort of getting ahead of the game here? It seems to me that your position has made a great many assumptions not even examined, much less accounted for.
Les Sleeth said:
I don’t know if you have discovered such an explanation, but I agree one is possible.
I have not discovered an explanation of anything! What I have discovered is that I can guarantee you that, for any explanation of anything, there exists a totally consistent interpretation of the symbols used to communicate that explanation which will require the fundamental elements of that explanation to obey my equation so long as that explanation is internally consistent. o:)

I think you should read that sentence over carefully in order to comprehend what I am saying. :bugeye:
Les Sleeth said:
In other words, if love has a physical aspect and a non-physical aspect, then the statement “love can be reduced to physics” means you think you can account for all aspects of love with physics (i.e., and not just that you are only accounting for the physical aspects). It is my recollection that you have said physics can cover it all many times.
No, that is not at all what I am saying. To paraphrase your comment, what I am saying is that, if "you" can account for all aspects of love in any internally consistent way, when you go to communicate your explanation to me, no matter how short or how long and involved that explanation might be, there will always exist a totally consistent interpretation of your explanation which will satisfy my equation. Paul, if you can explain that sentence to Les, please help me out. :redface:
Les Sleeth said:
Oh really? What evidence would that be? You above stated as plainly as one can that “my deduction is equivalent to observation.” That sir, unlike your claims, is unambiguous rationalism.
Somehow you have managed to miss the essence of my discourse. That every explanation of anything can be seen as a solution to my equation is a proved fact. That is an analytical fact flowing explicitly from my definition of an explanation. (An explanation, by the way, which you failed to criticize.) That fact has utterly nothing to do with reality and is merely a logical conclusion. The problem here is that all of physics may be shown to be a solution to that equation. The issue which arises through that profound difficulty is that, what the world considers observational evidence is exactly the defense of thousands of years of scientific work. The fact that their results are exactly the results of satisfying my equation constitutes an enormous body of evidence that my deduction is entirely equivalent to observation and that the rationalists are correct. :devil:

And finally, the fact that absolutely any explanation of anything can be interpreted to be a solution to my equation says that there is no fundamental difference between any explanation and the field of physics. The final nail in the coffin of your position is the fact that requiring the fundamental elements underlying your argument to obey the laws of physics says that what you are explaining is an emergent phenomena arising out of complexities of the interactions defined by those laws. Paul, if you aren't confused by that sentence, please help explain it to Les. :frown:

Have fun -- Dick
 
  • #174
Doctordick said:
.. It is that fact which answers the question, “Can everything be reduced to physics?” in the affirmative...
"Everything" ?? :confused: (1) Can "nothing" be explained by physics, since "nothing" is within the set of "everything" ? (2) Can metaphysics be explained by physics ? (3) Can a pure "effect" be discovered in the physics of its "cause" ? (4) What is the physics that explains the first movement in the universe ? (5) What is "good" according to the laws of physics ? (6) How does physics explain that the number 2 follows 1 ? (7) Can syllogism as a concept be reduced to physics ? I'll stop with these seven attempts to falsify your hypothesis--wait for answers.
 
  • #175
Rade said:
I'll stop with these seven attempts to falsify your hypothesis--wait for answers.
I am sorry. You simply did not understand what I said.
Doctordick said:
What I have discovered is that I can guarantee you that, for any explanation of anything, there exists a totally consistent interpretation of the symbols used to communicate that explanation which will require the fundamental elements of that explanation to obey my equation so long as that explanation is internally consistent. o:)
That is, you cannot give me a complete and internally consistent explanation of anything which can not be interpreted in a manner which makes it a solution to my equation. If you want to prove that statement false, you have only two choices. You may either find a specific error in my proof or you must provide me with a "complete and internally consistent explanation" which I cannot express as a solution to my equation: i.e., you must provide me with a way of determining exactly what your expectations are of the validity of any statement regarding of one of those things you mentioned. :rolleyes:

The second would be quite an achievement and probably worthy of a book even before I showed you how to properly represent the explanation (in fact, the easiest expression of your expectations would probably be a book in itself). The first would be far easier to achieve. :biggrin:

Have fun -- Dick
 

Similar threads

  • General Discussion
Replies
4
Views
1K
  • General Discussion
2
Replies
36
Views
5K
Replies
72
Views
10K
  • General Discussion
Replies
3
Views
3K
Replies
2
Views
2K
Replies
2
Views
2K
Replies
90
Views
6K
Replies
5
Views
4K
Replies
3
Views
3K
  • Quantum Interpretations and Foundations
Replies
8
Views
5K
Back
Top